Difference between revisions of "2023 AMC 8 Problems/Problem 12"

(Solution 2)
 
(38 intermediate revisions by 13 users not shown)
Line 1: Line 1:
==Animated Video Solution==
+
==Problem==
 +
The figure below shows a large white circle with a number of smaller white and shaded circles in its
 +
interior. What fraction of the interior of the large white circle is shaded?
 +
 
 +
<asy>
 +
// Diagram by TheMathGuyd
 +
size(6cm);
 +
draw(circle((3,3),3));
 +
filldraw(circle((2,3),2),lightgrey);
 +
filldraw(circle((3,3),1),white);
 +
filldraw(circle((1,3),1),white);
 +
filldraw(circle((5.5,3),0.5),lightgrey);
 +
filldraw(circle((4.5,4.5),0.5),lightgrey);
 +
filldraw(circle((4.5,1.5),0.5),lightgrey);
 +
int i, j;
 +
for(i=0; i<7; i=i+1)
 +
{
 +
draw((0,i)--(6,i), dashed+grey);
 +
draw((i,0)--(i,6), dashed+grey);
 +
}
 +
</asy>
 +
 
 +
 
 +
<math>\textbf{(A)}\ \frac{1}{4} \qquad \textbf{(B)}\ \frac{11}{36} \qquad \textbf{(C)}\ \frac{1}{3} \qquad \textbf{(D)}\ \frac{19}{36} \qquad \textbf{(E)}\ \frac{5}{9}</math>
 +
 
 +
==Solution 1==
 +
First, the total area of the radius <math>3</math> circle is simply just <math>9* \pi</math> when using our area of a circle formula.
 +
 
 +
Now from here, we have to find our shaded area. This can be done by adding the areas of the <math>3</math> <math>\frac{1}{2}</math>-radius circles and add; then, take the area of the <math>2</math> radius circle and subtract that from the area of the <math>2</math> radius 1 circles to get our resulting complex area shape. Adding these up, we will get <math>3 * \frac{1}{4} \pi + 4 \pi -\pi - \pi = \frac{3}{4} \pi + 2 \pi = \frac{11}{4}</math>.
 +
 
 +
So, our answer is <math>\frac {\frac{11}{4} \pi}{9 \pi} = \boxed{\textbf{(B)}\ \frac{11}{36}}</math>.
 +
 
 +
~apex304
 +
 
 +
==Solution 2==
 +
 
 +
Pretend each circle is a square. The large shaded circle is a square with area <math>16~\text{units}^2</math>, and the two white circles inside it each have areas of <math>4~\text{units}^2</math>, which adds up to <math>8~\text{units}^2</math>. The three small shaded circles become three squares with area <math>1~\text{units}^2</math>, and add up to <math>3~\text{units}^2</math>. Adding the areas of the shaded circles (19) and subtracting the areas of the white circles (8), we get <math>11~\text{units}^2</math>. Since the largest white circle in which all these other circles are becomes a square that has area <math>36~\text{units}^2</math>, our answer is <math>\boxed{\textbf{(B)}\ \dfrac{11}{36}}</math>.
 +
 
 +
-claregu
 +
LaTeX (edits -apex304, CoOlPoTaToEs)
 +
 
 +
==Video Solution by Math-X (How to do this question under 30 seconds)==
 +
https://youtu.be/Ku_c1YHnLt0?si=stUHQ9nHZZE_x-CC&t=1852 ~Math-X
 +
 
 +
 
 +
==Video Solution (HOW TO THINK CREATIVELY!!!) ==
 +
https://youtu.be/5wpEBWZjl6o
 +
 
 +
~Education the Study of everything
 +
 
 +
 
 +
==Video Solution (Animated)==
 
https://youtu.be/5RRo6pQqaUI
 
https://youtu.be/5RRo6pQqaUI
  
 
~Star League (https://starleague.us)
 
~Star League (https://starleague.us)
 +
 +
==Video Solution by Magic Square==
 +
https://youtu.be/-N46BeEKaCQ?t=4590
 +
 +
==Video Solution by SpreadTheMathLove==
 +
https://www.youtube.com/watch?v=UWoUhV5T92Y
 +
==Video Solution by Interstigation==
 +
https://youtu.be/DBqko2xATxs&t=1137
 +
 +
==Video Solution by harungurcan==
 +
https://www.youtube.com/watch?v=oIGy79w1H8o&t=1154s
 +
 +
~harungurcan
 +
 +
==See Also==
 +
{{AMC8 box|year=2023|num-b=11|num-a=13}}
 +
{{MAA Notice}}

Latest revision as of 14:33, 21 January 2024

Problem

The figure below shows a large white circle with a number of smaller white and shaded circles in its interior. What fraction of the interior of the large white circle is shaded?

[asy] // Diagram by TheMathGuyd size(6cm); draw(circle((3,3),3)); filldraw(circle((2,3),2),lightgrey); filldraw(circle((3,3),1),white); filldraw(circle((1,3),1),white); filldraw(circle((5.5,3),0.5),lightgrey); filldraw(circle((4.5,4.5),0.5),lightgrey); filldraw(circle((4.5,1.5),0.5),lightgrey); int i, j; for(i=0; i<7; i=i+1) { draw((0,i)--(6,i), dashed+grey); draw((i,0)--(i,6), dashed+grey); } [/asy]


$\textbf{(A)}\ \frac{1}{4} \qquad \textbf{(B)}\ \frac{11}{36} \qquad \textbf{(C)}\ \frac{1}{3} \qquad \textbf{(D)}\ \frac{19}{36} \qquad \textbf{(E)}\ \frac{5}{9}$

Solution 1

First, the total area of the radius $3$ circle is simply just $9* \pi$ when using our area of a circle formula.

Now from here, we have to find our shaded area. This can be done by adding the areas of the $3$ $\frac{1}{2}$-radius circles and add; then, take the area of the $2$ radius circle and subtract that from the area of the $2$ radius 1 circles to get our resulting complex area shape. Adding these up, we will get $3 * \frac{1}{4} \pi + 4 \pi -\pi - \pi = \frac{3}{4} \pi + 2 \pi = \frac{11}{4}$.

So, our answer is $\frac {\frac{11}{4} \pi}{9 \pi} = \boxed{\textbf{(B)}\ \frac{11}{36}}$.

~apex304

Solution 2

Pretend each circle is a square. The large shaded circle is a square with area $16~\text{units}^2$, and the two white circles inside it each have areas of $4~\text{units}^2$, which adds up to $8~\text{units}^2$. The three small shaded circles become three squares with area $1~\text{units}^2$, and add up to $3~\text{units}^2$. Adding the areas of the shaded circles (19) and subtracting the areas of the white circles (8), we get $11~\text{units}^2$. Since the largest white circle in which all these other circles are becomes a square that has area $36~\text{units}^2$, our answer is $\boxed{\textbf{(B)}\ \dfrac{11}{36}}$.

-claregu LaTeX (edits -apex304, CoOlPoTaToEs)

Video Solution by Math-X (How to do this question under 30 seconds)

https://youtu.be/Ku_c1YHnLt0?si=stUHQ9nHZZE_x-CC&t=1852 ~Math-X


Video Solution (HOW TO THINK CREATIVELY!!!)

https://youtu.be/5wpEBWZjl6o

~Education the Study of everything


Video Solution (Animated)

https://youtu.be/5RRo6pQqaUI

~Star League (https://starleague.us)

Video Solution by Magic Square

https://youtu.be/-N46BeEKaCQ?t=4590

Video Solution by SpreadTheMathLove

https://www.youtube.com/watch?v=UWoUhV5T92Y

Video Solution by Interstigation

https://youtu.be/DBqko2xATxs&t=1137

Video Solution by harungurcan

https://www.youtube.com/watch?v=oIGy79w1H8o&t=1154s

~harungurcan

See Also

2023 AMC 8 (ProblemsAnswer KeyResources)
Preceded by
Problem 11
Followed by
Problem 13
1 2 3 4 5 6 7 8 9 10 11 12 13 14 15 16 17 18 19 20 21 22 23 24 25
All AJHSME/AMC 8 Problems and Solutions

The problems on this page are copyrighted by the Mathematical Association of America's American Mathematics Competitions. AMC logo.png